LSAT and Law School Admissions Forum

Get expert LSAT preparation and law school admissions advice from PowerScore Test Preparation.

 Administrator
PowerScore Staff
  • PowerScore Staff
  • Posts: 8950
  • Joined: Feb 02, 2011
|
#23505
Complete Question Explanation

Must be True. The correct answer choice is (A)

Although technically the answer to this question does not have to be true, this is still a Must Be True question and is approached the same way. From the stimulus, we know that the current piecemeal system is not working, cannot provide any coherent solutions, and gives health care providers and insurers every incentive to shift health care costs upon patients or each other. So, the question to ask ourselves her is what conclusion results naturally from that information. If you were to prephrase an answer here, it would probably have something to do with the fact that a more complete plan in necessary.

Answer Choice (A): This is the correct answer choice. The stimulus tells us outright that solutions to the problem cannot be found in the current piecemeal system. Therefore, it follows that the only possible means of reducing health care costs is through a more comprehensive approach.

Answer Choice (B): There is nothing in the stimulus addressing the income of health care professionals, and thus there is nothing in the stimulus that we can look at to support this answer choice.

Answer Choice (C): Again, since there is nothing in the stimulus regarding the expansion of health care funds relative to health care costs, we can find no support for this answer choice.

Answer Choice (D): The stimulus does not address any technological issues or any costs attributed to technology. Once again, there is nothing in the stimulus to support this answer choice.

Answer Choice (E): One last time, since there is nothing in the stimulus addressing anything close to the issue in this answer choice, there is nothing to support this answer choice.
 reop6780
  • Posts: 265
  • Joined: Jul 27, 2013
|
#10172
Unfortunately, I lack the full understanding of this stimulus especially the last sentence, "For example, ~ patients with advanced illness later presented themselves at hospital emergency rooms in increased numbers." :-?

From what I understand, the stimulus describes the weakness of the current health-care policy. Even though the policy tries to reduce the cost for patients, physicians will receive the remaining cost that they did not receive from some patients by other patients with more serious illness.

However, I do not understand how disallowing payments from some patients lead to the increase of patients with advanced illness.

(*Is it necessary to understand the logic used by the author to find the correct answer? )

Also, if I change "unwise" to "wise" in the answer E, is the answer E qualified as the right answer ?

(* I believe the first part of "Since~ patient's bill" is correct)

Thank you

Hyun Kim
 David Boyle
PowerScore Staff
  • PowerScore Staff
  • Posts: 836
  • Joined: Jun 07, 2013
|
#10211
reop6780 wrote:On page 31, question number 37, the answer is A.

Unfortunately, I lack the full understanding of this stimulus especially the last sentence, "For example, ~ patients with advanced illness later presented themselves at hospital emergency rooms in increased numbers." :-?

From what I understand, the stimulus describes the weakness of the current health-care policy. Even though the policy tries to reduce the cost for patients, physicians will receive the remaining cost that they did not receive from some patients by other patients with more serious illness.

However, I do not understand how disallowing payments from some patients lead to the increase of patients with advanced illness.

(*Is it necessary to understand the logic used by the author to find the correct answer? )

Also, if I change "unwise" to "wise" in the answer E, is the answer E qualified as the right answer ?

(* I believe the first part of "Since~ patient's bill" is correct)

Thank you

Hyun Kim
Dear Hyun:

Yes, it's usually good to understand the logic used by the author. It can be hard to find the correct answer without that.
"...patients with advanced illness later presented themselves at hospital emergency rooms in increased numbers" likely means that since those patients couldn't get treatment normally, then those patients either got sicker and had more emergencies, or figured that the doctors in the emergency room had to accept them even without money, or both.
Answer choice E just sounds cruel even if you put "wise" in it, and I doubt that's what the author's talking about. (He or she isn't trying to cut emergency care, just to discuss how bad piecemeal health reform is.) Since the author pooh-poohs piecemeal reform, he probably wants more comprehensive reform--thus answer choice A.

David
 co659
  • Posts: 10
  • Joined: Apr 23, 2017
|
#38103
Hello!

I had trouble with this problem because the first line of the stimulus states "Coherent solutions for the problem of reducing health-care costs cannot be found within the current piecemeal system of paying these costs." The author then continues to provide reasoning as to why they posited this statement (i.e. shifting costs, hospital # example.)

The question is asking us to strenghten what is already said in the stimulus so I'm confused as to why (A) is correct when the author already stated that there is no comprehensive solution in the current system?
 Eric Ockert
PowerScore Staff
  • PowerScore Staff
  • Posts: 164
  • Joined: Sep 28, 2011
|
#38490
Hi co!

Careful. This isn't a Strengthen question, it is a Must Be True question. Here we are using the argument in the stimulus to support one of the answer choices, not the other way around.

You are right that the author concludes that comprehensive solutions cannot be found in the current piecemeal system. But if that is the case, the only solution to healthcare costs must be found outside of this system. And that is exactly what answer (A) is saying. So (A) is provable based on the statements in the stimulus, and therefore correct as a Must Be True answer.

Hope that clears it up a bit!
 lunsandy
  • Posts: 61
  • Joined: Oct 14, 2017
|
#41592
Hi Powerscore!

Upon review I chose A, but I am still struggling to see why E is incorrect. I chose E initially because I thought "unfilled hospital beds contribute to overhead charges and unwise to hold unused capacity for large emergencies" aligns with the stim saying that is is problematic to shift cost from A to B, or if A is not solved then B accumulates to something worse.

I struggled to see A as the correct answer because how are we suppose know that if piecemeal system is not a good approach than the only other option we have is a comprehensive approach? Can't we have something that is both piecemeal and comprehensive? I guess I just didn't see it as two options only.

Thanks a lot!
 James Finch
PowerScore Staff
  • PowerScore Staff
  • Posts: 943
  • Joined: Sep 06, 2017
|
#41600
Hi Lunsandy,

As a Must be True question, the correct choice will always flow directly from the premises. This stimulus differs from more modern tests in that we already have a conclusion given to us in the stimulus, with a set of premises. Given that structure, we are looking for a statement that would be supported by the conclusion given in the stimulus (which would then become an intermediate conclusion in our argument.

So only (A) is supported by the conclusion we have in the stimulus ("Coherent solutions for the problem of reducing health-care costs cannot be found within the current piecemeal system of paying these costs."). (E) presents an entirely separate, self-contained argument, with both premise and conclusion, about a separate topic (holding hospital beds in reserve for large-scale emergencies). It does not link up in any way to the stimulus, especially not the conclusion within it, which deals with reducing health-care costs.

Hope this clears things up!
 mjb514
  • Posts: 35
  • Joined: Nov 01, 2017
|
#41940
How can one tell this is a must be true question rather than a strengthen question?
 Francis O'Rourke
PowerScore Staff
  • PowerScore Staff
  • Posts: 471
  • Joined: Mar 10, 2017
|
#42090
Hi mjb,

The question stems asks us which answer choice the argument provides support for. In other words, we are being asked to use the information in the stimulus to support, or prove, some statement among the five answers.

Compare this to a Strengthen question. A question stem of this type might ask you to find an answer choice that strengthens the argument in the stimulus. The flow of the information is the reverse of a Must Be True question. instead of using the information above to support an answer choice, you are using the answer choice to support the conclusion in the stimulus.

You can also expect strengthen questions to tell you to assume each answer choice is true, while you evaluate how they would impact the reasoning in the stimulus.
 whardy21
  • Posts: 48
  • Joined: Sep 30, 2018
|
#65958
I see that there is a slight difference in most strongly be supported and must be true. I was between A and C. If I apply the fact test to A, we don't know what could be if we apply the approach. That is not stated in the stimulus, however logical sense would tell you this is only approach to fix the problem i.e. most strongly be supported.

Get the most out of your LSAT Prep Plus subscription.

Analyze and track your performance with our Testing and Analytics Package.